A filter in the shape of a cone has a diameter of 3 inches and a height of 4 inches.How much liquid can the filter hold without it overflowing?
A) π in³
B) 3π in³
C)12π in³
D)36 π in³
please answer this Thank you

Answers

Answer 1

[tex] \huge ༆ \bf{Answer} ༄[/tex]

The liquid that the cone can store is its volume, So let's calculate the volume of cone

It is given by : -

[tex] \sf \: v = \dfrac{1}{3} \pi {r}^{2} h[/tex]

radius = diameter ÷ 2 = 3 ÷ 2 = 1.5 in

height = 4 inches

[tex] \sf \: v = \dfrac{1}{3} \pi \times 1.5 \times 1.5 \times 4[/tex]

[tex] \sf \: v = \pi \times 1.5 \times 2[/tex]

[tex] \sf \: v = 3\pi \: \: in {}^{3} [/tex]

The liquid that the cone can hold without overflowing is 3π in³

[tex]꧁ \: \frak{Eternal \: Being } \: ꧂[/tex]


Related Questions

Help please ! I need the answer ASAP

Answers

Answer:

[tex](f + g)(1) = - 3[/tex]

Step-by-step explanation:

[tex](f + g)(x ) = (2x - 3) + (4x - 6) \\ = 6x - 9 \\ \\ (f + g)(1) = 6(1) - 9 \\ = 6 - 9 \\ = - 3[/tex]

I hope I helped you^_^

please help, I don't know what to do​

Answers

Answer:

6%

Step-by-step explanation:

60 = 500•2•rate

60 = 1000•rate

rate = 60/1000

rate = 0.06 = 6%

simplify root 12 over root 3

Answers

Answer:

4

3

Step-by-step explanation:

pls mark brainliest if possible

10x2 + 31x + 24
(2x + 3)(5x + 8)

Answers

Is it right? Nnnnnnn

A right triangle is one that contains a ______ degree angle​

Answers

Answer:

90 degrees

Step-by-step explanation:

......

Answer: 90 degree angle.

Step-by-step explanation: A right triangle must have a right angle.

How to Solve 2x = 9?

Answers

=》2x = 9

=》x = 9/2

=》x = 4.5

■ Take all the numbers to the right side leaving the variable alone on the the other side of the equation & then simplify it.

_____

RainbowSalt2222 ☔

Any help would be appreciated

Answers

Answer:

Q1: 0.88015 ~ 88.015%, are

Q2: 0.88176 ~ 88.176%, are not

Step-by-step explanation:

Okay so a simple way to do these probability ones is to add an extra space around each side  

                                   A        B        C        D

ACCURATE               310    262    246    144    ___

NOT ACCURATE       31       51       37       12    ___

                                  ___    ___     ___    ___   ___  

Now in these spaces add all the the rows and columns accordingly  

                                    A          B          C          D

ACCURATE               310      262      246      144       _962_

NOT ACCURATE       31         51         37         12        _131_

                                 _341_  _343_  _283_  _156_   _1093_

Now the number in the place where the additional rows meet is your total number of orders, so this is what you would use for your probability.  

So we are looking for ACCURATE / ALL which is 962  

Now do 962 divided by 1093 and your probability comes out to 0.88015, so as a percentage, would be 88.015%

With replacement, all variables become independent, however without replacing that changes the probability.

When the 2 orders are selected, the final number of probability changes - so it would take down the total each time, the find the mean or average of the probability for your answer.

So since there is no replacement for ACCURATE / ALL which is 962, the division instead would be by 1091, since we are taking 2 orders out and not putting them back, it brings down the total number by 2, which would make the probability come out to 0.88176, or 88.176%

will mark brainly :)
y=1/2(x-3)^2
y=1/2(x+3)^2
y=2(x-3)^2
y=2(x+3)^2

Answers

Answer:

y = 1/2 (x-3) ^2

Step-by-step explanation:

I hope this helps!

YES I AM GVING AN BRAINLIST = ACCURATE
Match each pair of graphs to a situation.

Answers

Answer:

d: 2

e: 1

f: 3

If you need explanation just ask i can give the reason :)

Step-by-step explanation:

HELP1!!1!!1!


Solve your word problem using a model. Use complete sentences to describe your model, and interpret the quotient in the context of your word problem. If you wish, you may upload a copy of your model.

Answers

The model [tex]x = \frac{6}{3}[/tex], can be used to solve the world problem representing a scenario where John wants to share 6 apples amongst his three friends. x = 2, is the quotient which is interpreted as the number of apples each of John's friend will get.

Word problem describes real world situations which can be translated into expressions as a model.

For example, if John has 6 apples and John has three friends, how many apples would each of the friend get (x) if he is to share the 6 apples amongst the three of them?

To solve this, we can model the situation as follows:

Let x represent the number of apples each friend would get (quotient).

Thus:

[tex]x = \frac{6}{3}\\\\\mathbf{x = 2}[/tex]

The quotient (x) in this model can be interpreted as the number of apples each of John's friend will get.

Thus, the model [tex]x = \frac{6}{3}[/tex], can be used to solve the world problem representing a scenario where John wants to share 6 apples amongst his three friends. x = 2, is the quotient which is interpreted as the number of apples each of John's friend will get.

Learn more here:

https://brainly.com/question/13818690

simplify the expression below and rewrite it in rational exponent form

Answers

Well its D. 5x^9/10.y^1/3

if <mac=86 of a pizza and I ate it, how many degrees of pizza did I give to you​

Answers

A pizza consists of a whole circle i.e 360°

So

Let your part be x

[tex]\\ \rm\Rrightarrow x+86=360[/tex]

[tex]\\ \rm\Rrightarrow x=360-86[/tex]

[tex]\\ \rm\Rrightarrow x=274°[/tex]

A soccer game is 90 minutes long. 36 minutes have passed. What percentage of the game has passed?​

Answers

Answer:

The answer is 40%

Step-by-step explanation:

Answer:

40% increase

Step-by-step explanation:

90 - 36 = 54

90 - 54 = 36

36 / 90 = 0.4

0.4 x 100 = 40%

graphically represent
The cuadratic relation

Answers

boberson is after then my mom

Which of the following must be true?

Answers

Answer:

answer is B because in the property of parallelogram it says ad is parallel to bc

78 decreased by half a number is 45 Find the number

Answers

78 - 45 = 33
Since it’s half a number you do
33 x 2 = 66
So the answer is 66

Answer:

number is 66

Step-by-step explanation:

let the number be n , then

78 - [tex]\frac{1}{2}[/tex] n = 45 ( subtract 78 from both sides )

- [tex]\frac{1}{2}[/tex] n = - 33 ( multiply both sides by - 2 to clear the fraction )

n = 66

The required number is 66

Pls help, will give brainliest if your answer is right!

Question in image.

Answers

Step-by-step explanation:

Mean = Average : Sum of the data ÷ Number of data

So find the average number of lates per student

4 = Lates

30 (sum of frequencies) = Frequencies

So 4 ÷ 30 = 7.5

Therefore the answer is 7.5..

I WILL GIVE BRAINIEST, PLEASE HELP

Answers

Answer:

A = (1, 10)

B = (4,10)

C = (4, 14)

D = (1, 16)

Step-by-step explanation:

Okay, so first, we move, or translate the quadrilateral PQRS down -8 spaces on the graph, turning old points:

P (1, -2) = (1, -10)     Q (4, -2) = (4, - 10)     S (1, -8) = (1, -16)     and R (4, -6) = (4, -14)

Now, we need to rotate this new box into -x, y. So our new points are:

(1, -10) = (-1, 10)     (4, -10) = (-4, 10)     (1, -16) = (-1, 16)     and (4,-14) = (-4, 14)

Now we're reflecting off the y-axis, making our x change. So for our final points:

A(1, 10)   B (4,10)    D(1, 16)    and C(4, 14)

Hope this helps! Please check my math and comment wether this worked or not! :)

Answer:

hi

Step-by-step explanation:

A coin was tossed 80 times, out of which it landed on heads 60 times. What is the relative frequency of the coin toss resulting in tails?

Answers

Answer : 25%

Step-by-step explanation : If a coin was tossed 80 times and it landed 60 times, 80 - 60 = 20 so 20/80 were tails, let's convert that into % so 20 is 1/4 of 80 so 25%.

Factoring polynamials: 6x^2y-3xy^2

Answers

Answer:

 -y • (6x^2 + 3xy - 4)

Step-by-step explanation:

what is the answer, i tried everything but i can’t seem to get the right answer

Answers

Answer:A

Step-by-step explanation: It never talks about 50 so i don't know why that is there so I think it is A because 21 dollars is more than 5+2.75, (not 100% sure but at least 85% sure)

Answer:

A

Step-by-step explanation:

The reason that this is A is because 5$ is the initial fee and you dont pay any more than that, since 21$ is the max you can afford it is going to be less than or equal to 21. and if it is 2.75$ per stop then it is the number of stops times 2.75. A is your answer.

edit: and the 50 kilometers doesnt have any value in the situation so basically it was just there to confuse you or throw you off.

Colleen buys a robot priced at $33.If the sales tax is10%,how much tax will Colleen pay?

Answers

Answer:

$3.3 tax

Final price is 36.3

Step-by-step explanation:

Firstly, if the tax is expressed in percent, divide the tax rate by 100. You can do this by simply moving the decimal point two spaces to the left. In this example, we do 10/100 = 0.1

tax= 33 * 0.1

tax=3.3

Finally, add tax to the before tax price to get the final price including tax.

The final price including tax = 33 + 3.3 = 36.3


Consider this function.
h(x) = (x - 2)^2+3

Which of the following domain restrictions would enable h(x) to have an inverse function?
a. x < 1
b. x >5
c. x < 3
d. x > 4

(Ps: all four answer and larger equal then or smaller equal then

Answers

Answer:

No inverse function:  (a), (b), (c)

Inverse function exists:  (d)

Step-by-step explanation:

The graph of h(x) = (x - 2)^2 + 3 is a parabola that opens upward and has vertex at (2, 3).  If the entire graph is drawn, and the horizontal line test then applied, h(x) would not have an inverse, because the horizontal line would intersect the  parabolic graph twice.

Note that if we restricted the domain to x ≥ 2, the resulting graph would pass the horizontal line test.  This would also be true for x ≥ 3, x ≥ 4, and so on.  Not so for (a) x < 1.  False for x > -5.  True for x < 3.  True for x > 4.

No inverse function:  (a), (b), (c)

Inverse function exists:  (d)

Your friend tried putting away the sugar for you and cannot get the 14 cups of sugar to fit in the containers! They determined you would need 9 containers to store the 14 cups of sugar.

They show you their calculations:



Answer the following questions.

What mistake did your friend make in calculating the number of containers needed?
*Hint* Look closely at the work shown to find the mistake

How many containers are needed to store the leftover sugar?

Answers

Answer:

21 containers

Step-by-step explanation:

When dividing fractions, you can multiply by the inverse of the fraction. For example, in this case, [tex]14 divided by \frac{2}{3} = \frac{14}{1}x\frac{3}{2}[/tex]. Whoever drew the calculations made the mistake of forgetting to flip the second fraction.

The actual solution of the problem is 42/2, or 21/1. 21 containers are needed to store the sugar.

Three sisters have ages that are consecutive odd integers. The sum of the age of the youngest and three times the age of the oldest is five less than five times the middle sister’s age.

Answers

Answer:

youngest   17  

middle  19

oldest    21

Step-by-step explanation:

Three sisters have ages that are consecutive odd integers

youngest = x

middle = x + 2

oldest = x + 4

--------------------

The sum of the age of the youngest

x

and three times the age of the oldest

+ 3(x + 4)

is five less than five times the middle sister’s age

= 5(x + 2) - 5

-----------------------

x + 3(x + 4) = 5(x + 2) - 5

Distribute

x + 3x + 12 = 5x + 10 - 5

Combine like terms

4x + 12 = 5x - 5

Subtract 4x from both sides

12 = x - 5

Add 5 to both sides

17 = x

youngest   x = 17  

middle     x + 2 = 19

oldest     x + 4 = 21

please help me i don't understand

Answers

Answer:

D and A

Step-by-step explanation:

1. 610:488 simplify

5:4

2.360:(488+551+610+360+391)

360:2400

3:20

Pleaaasee please helllpppp

Answers

Answer:

iodine gold mercury

Step-by-step explanation:

Heyyyyyyyyyyyyyyyyyyyyyy

Answers

Answer:

7/12

Step-by-step explanation:

you have to find a common denominator, which would be 12. it would be 4/12 + 3/12. add them up, and you get 7/12

1/3 + 1/4 =

4/12+3/12= 7/12

The width of a rectangular parking lot at the meadowlands is 3/8 mile. If the area is 5/8 square mile, what is the length of the parking lot

Answers

let width is x and length is y

x * y = area

=> 3/8 * y = 5/8

=> y = 5/8 ÷ 3/8

=> y = 5/8 * 8/3 = 5/3

the length of the rectangular parking lot is 5/8 mile.

Please help!!! I have to submit this by today.

[tex]i \div 8 = 12[/tex]

Answers

Answer:

i = 96

Step-by-step explanation:

i ÷ 8 = 12i = 12 × 8i = 96
Other Questions
On Monday, 339 students went on a trip to the zoo. All 6 buses were filled and 9 students had to travel in cars. How many students werein each bus ? What was Frederick Douglass main complaint to President Abraham Lincoln? Please help Im so lost!! Chemistry Context Clues: The Meaning Is There Write the equation of the line with x-intercept 3 and passing through the point (5.4) At a gathering of 15 people, there are 10 people who all know each other and 5 people who know no one. People who do not know each other shake hands. How many handshakes occur? what is chemical reaction? Name four conditions required for chemical reaction with an examples of each. Find the midpoint of the line segment joining the points (-1,-1) and (-3,10). Let's Explore This Suggested Time Allotment: 15 Minutes Answer the questions in each column. What could be the result of your action? How will you respond Situation to show respect for individual differences in your family? 1. Your sibling is answering the school modules when you realized that it was already time to wash the dishes. Last time when it was your turn, you stopped doing your school work and did household chores. 2. Your parents promised that they will buy a new mobile phone for you. However, because of the pandemic, they decided to buy a personal computer that you can share with your other siblings instead 3. Your siblings like to watch their favorite TV show while you are studying. You try to explain to them that you cannot concentrate if they would turn on the television but they insist on watching. ALU 1 agkatareal answer=brainlistnonsense=spam find an equation for the line below when ben & jerrys ice cream tests all its proposed new flavors on its own employees at its corporate headquarters in vermont, it is partaking in Danielle found an outfit she liked at two different stores. Store 1 marked the outfit down by 15%. They then marked down the reduced price an additional 5% off. Danielle had a 20% off coupon for Store 2. If the outfit originally cost $85 at both stores, which store is better buy for Danielle? a=V^2/P make v the subject how is mobility issues a problem to attend a health care service Question 25Anti-Federalist leaders: a. tended to be younger than their Federalist counterpartsb. often were better organized and prepared than their Federalist opponentsc. had been the chief proponents of a stronger central government at the Constitutional Conventiond. tended to be wealthier than their Federalist opponents lm r lin c nhn '' lin c nhn l g'' qua 2 kiu quan h quyn lc v thn hu trong l lun vn hc Pls help I am desperate hhhhhhhhhhheeeeeeeeeeeellllllllllpppppppppppppp!!!!!!!!!!!!!!!! Write the ratio as a fraction in simplest form 2 5/7 to 1 1/7 The equation y = kx represents the relationship between x and y in this table.x y13 918 5629 20331 217What is the value of k?